Help
I am struggling with this question because I don't understand why it can't be 7. The restrictions...
Sendi on July 23, 2022
  • June 2007 LSAT
  • SEC1
  • Q8
5
Replies
HELP
Why can't Thursday be LGH? The days that have restrictions are Friday and Saturday as they cannot...
Yanela on May 1, 2022
  • June 2007 LSAT
  • SEC1
  • Q8
1
Reply
LSAT
Done
01718935113 on April 15, 2022
  • June 2007 LSAT
  • SEC1
  • Q9
1
Reply
LSAT
Done
01718935113 on April 15, 2022
  • June 2007 LSAT
  • SEC1
  • Q9
1
Reply
LSAT
Done
01718935113 on April 15, 2022
  • June 2007 LSAT
  • SEC1
  • Q9
1
Reply
Video Explanation
Hi. Do the reading comp sections not have video explanations? They were really helpful
maonuo on March 26, 2022
  • June 2007 LSAT
  • SEC4
  • Q1
3
Replies
June 2007 test
If I don’t do well on this test does that determine anything
Rhena on March 22, 2022
  • June 2007 LSAT
  • SEC2
  • Q3
5
Replies
Error
I was reviewing my answers and there are two points were my answer was changed from correct to a ...
juma on March 18, 2022
  • June 2007 LSAT
  • SEC3
  • Q9
2
Replies
Conditions
I’m not sure how on this question it was assumed that if you had a newspaper you could also a pai...
CJLEAL on March 9, 2022
  • June 2007 LSAT
  • SEC1
  • Q21
1
Reply
Video explains Tio needs
The video explanation goes a little bit too fast on this one for me is there anyone else could ge...
CJLEAL on March 9, 2022
  • June 2007 LSAT
  • SEC1
  • Q19
1
Reply
Setup
I’m confused as why there can’t be all three fils shown in one day during a review of these quest...
CJLEAL on March 9, 2022
  • June 2007 LSAT
  • SEC1
  • Q8
1
Reply
Question Assistance
Why isn't B correct?
drueyoung on February 12, 2022
  • June 2007 LSAT
  • SEC1
  • Q11
2
Replies
Question about the paragraph
Isn’t one of the given facts in the paragraph that each film can show NO MORE THAN ONCE in given ...
mari2 on February 8, 2022
  • June 2007 LSAT
  • SEC1
  • Q8
3
Replies
Question RC
Can someone please explain the correct answer
maonuo on February 8, 2022
  • June 2007 LSAT
  • SEC4
  • Q11
21
Replies
Scenarios
Hello!! So I am curious as to how you knew there was another scenario that could be developed ...
Mari on February 7, 2022
  • June 2007 LSAT
  • SEC1
  • Q9
5
Replies
help
I picked D, I felt that was a flaw in the argument
chriscole on February 5, 2022
  • June 2007 LSAT
  • SEC3
  • Q25
8
Replies
I didn't pick B because of "most".. please help...
Hey there, I didn't pick B because of the term "most" because I thought couldn't a "little" only...
Ryn on January 19, 2022
  • June 2007 LSAT
  • SEC2
  • Q5
2
Replies
Abstract Principle?
How do we know what Jane based her decision on was an abstract principle?
Ryn on January 19, 2022
  • June 2007 LSAT
  • SEC2
  • Q7
1
Reply
still dont get the explanations
G-T-M-G-M-J-T I think for A to be the answer then above has to be the sequence. I think the co...
medasmx@protonmail.com on January 14, 2022
  • June 2007 LSAT
  • SEC1
  • Q15
2
Replies
Why can't it be C?
I'm confused in part of the explanation. Where in the rules does it say that there has to be glas...
terrierbl41 on August 24, 2021
  • June 2007 LSAT
  • SEC1
  • Q23
4
Replies